LSAT and Law School Admissions Forum

Get expert LSAT preparation and law school admissions advice from PowerScore Test Preparation.

 Administrator
PowerScore Staff
  • PowerScore Staff
  • Posts: 8916
  • Joined: Feb 02, 2011
|
#26379
Complete Question Explanation
(The complete setup for this game can be found here: lsat/viewtopic.php?t=7717)

The correct answer choice is (C)

M can only appear first under Template #1. In that template J appears sixth, and so answer choice (C) is correct.
 cmnoury1221
  • Posts: 21
  • Joined: Sep 11, 2019
|
#71744
Hello,
Why is answer choice A wrong? Is it because G can be 4th?
M-L-H-G-P-J ?

Thanks!
carolyn
 Rachael Wilkenfeld
PowerScore Staff
  • PowerScore Staff
  • Posts: 1358
  • Joined: Dec 15, 2011
|
#71773
Hi cmnoury1221,

Yes, in this question, we are looking for what is absolutely required. Once we know M is first, we are in the first template (the templates and set up are linked in the first post on this thread). The only other thing that is certain once M is in 1 is that J has to be last. G could be 4th or 5th. P can float between slots 2 and 5 as it would keep it before J but after M. That means that we can't be certain about who goes in any of the slots 2 through 5 in this question because P is so underdetermined.

Hope that helps!
Rachael

Get the most out of your LSAT Prep Plus subscription.

Analyze and track your performance with our Testing and Analytics Package.